LSAT and Law School Admissions Forum

Get expert LSAT preparation and law school admissions advice from PowerScore Test Preparation.

 Administrator
PowerScore Staff
  • PowerScore Staff
  • Posts: 8929
  • Joined: Feb 02, 2011
|
#40231
Complete Question Explanation
(The complete setup for this game can be found here: lsat/viewtopic.php?t=8561)

The correct answer choice is (D)

If the Yandells owned M, we need to conform to the 2-2-1 distribution (Templates 1A and 2A). In a 1-3-1 distribution, M cannot be assigned to the Yandells, because the second rule would require us to split F and I between the remaining two groups. As a result, neither the Yandells would own I, nor the Trents would own S, in direct violation of the third rule.

Since Templates 1A and 2A represent the only viable solutions to this question, we can modify each template in accordance with the new condition:
PT73_Game_#3_#16_diagram 1.png
In both solutions, the Williamses own G—a prephrase that agrees with answer choice (D).

Answer choice (A) is incorrect, because the Trents did not necessarily own F, as shown in Template 1A.

Answer choice (B) is incorrect, because the Trents did not necessarily own I, as shown in Template 2A.

Answer choice (C) is incorrect, because the Williamses did not necessarily own F, as shown in Template 2A.

Answer choice (D) is the correct answer choice, since the Williamses own G in both solutions.

Answer choice (E) is incorrect, because the Williamses did not necessarily own I, as shown in Template 1A.
You do not have the required permissions to view the files attached to this post.

Get the most out of your LSAT Prep Plus subscription.

Analyze and track your performance with our Testing and Analytics Package.